Quantcast
  • Register
PhysicsOverflow is a next-generation academic platform for physicists and astronomers, including a community peer review system and a postgraduate-level discussion forum analogous to MathOverflow.

Welcome to PhysicsOverflow! PhysicsOverflow is an open platform for community peer review and graduate-level Physics discussion.

Please help promote PhysicsOverflow ads elsewhere if you like it.

News

PO is now at the Physics Department of Bielefeld University!

New printer friendly PO pages!

Migration to Bielefeld University was successful!

Please vote for this year's PhysicsOverflow ads!

Please do help out in categorising submissions. Submit a paper to PhysicsOverflow!

... see more

Tools for paper authors

Submit paper
Claim Paper Authorship

Tools for SE users

Search User
Reclaim SE Account
Request Account Merger
Nativise imported posts
Claim post (deleted users)
Import SE post

Users whose questions have been imported from Physics Stack Exchange, Theoretical Physics Stack Exchange, or any other Stack Exchange site are kindly requested to reclaim their account and not to register as a new user.

Public \(\beta\) tools

Report a bug with a feature
Request a new functionality
404 page design
Send feedback

Attributions

(propose a free ad)

Site Statistics

205 submissions , 163 unreviewed
5,054 questions , 2,207 unanswered
5,347 answers , 22,720 comments
1,470 users with positive rep
818 active unimported users
More ...

  Matrix element of the derivative of an operator between its eigenstates

+ 1 like - 0 dislike
671 views

I want to calculate a matrix element of the derivative of the Hamiltonian between two eigenstates $\alpha$ and $\beta$ given by $u^\alpha(x,y)$ and $u^\beta(x,y)$ (called the Bloch functions): $$\langle \alpha | \frac{\partial \hat{H}}{\partial k_j}|\beta\rangle$$This is taken from Eq. (3.6) in the paper by Komoto, Topological Invariant and the Quantization of the Hall conductance. In the same paper in Eq. (3.4), they defined the matrix element of v between states $\alpha$ and $\beta$ as:

$$(v)_{\alpha \beta}=\delta_{k_1 k'_1} \delta_{k_2 k'_2} \int_0^{qa} dx \int_0^b dy u_{k_1 k_2}^{\alpha^*} v u_{k'_1 k'_2}^{\beta}$$

The result from the paper is: $(E^\beta -E^\alpha)\langle \alpha| \frac{\partial u^\beta}{\partial k_j} \rangle=-(E^\beta - E^\alpha)\langle \frac{\partial u^\alpha}{\partial k_j}| \beta\rangle$.

I tried using the expression for matrix element between states given in the paper but cannot obtain their result. I think there has to be an integration by parts involved in order to get $(E^\beta - E^\alpha)$ but integration by parts requires the presence of an integral over $k_j$ which is not the case here.

asked Jan 2, 2018 in Theoretical Physics by zahmad (5 points) [ no revision ]

1 Answer

+ 0 like - 0 dislike

Consider the general matrix element $E^{\beta}\langle \alpha |\nabla \beta \rangle$. This can be simplified as: $$E^{\beta}\langle \alpha |\nabla \beta \rangle=\langle \alpha |\nabla (\hat{H} \beta) \rangle=\langle \alpha |\nabla \hat{H}| \beta \rangle + \langle \alpha |\hat{H} \nabla \beta \rangle=\langle \alpha |\nabla \hat{H}| \beta \rangle + E^{\alpha}\langle \alpha |\nabla \beta \rangle$$

Hence, $\langle \alpha |\nabla \hat{H}| \beta \rangle=(E^{\beta} - E^{\alpha})\langle \alpha |\nabla \beta \rangle$. Similarly if one repeats the calculation starting from $E^{\alpha}\langle \beta |\nabla \alpha \rangle$, one gets $\langle \alpha |\nabla \hat{H}| \beta \rangle=-(E^{\beta} - E^{\alpha})\langle \nabla \alpha | \beta \rangle$. This is exactly what is required to be proved.

answered Apr 2, 2018 by zahmad (5 points) [ no revision ]

Your answer

Please use answers only to (at least partly) answer questions. To comment, discuss, or ask for clarification, leave a comment instead.
To mask links under text, please type your text, highlight it, and click the "link" button. You can then enter your link URL.
Please consult the FAQ for as to how to format your post.
This is the answer box; if you want to write a comment instead, please use the 'add comment' button.
Live preview (may slow down editor)   Preview
Your name to display (optional):
Privacy: Your email address will only be used for sending these notifications.
Anti-spam verification:
If you are a human please identify the position of the character covered by the symbol $\varnothing$ in the following word:
p$\hbar$ysi$\varnothing$sOverflow
Then drag the red bullet below over the corresponding character of our banner. When you drop it there, the bullet changes to green (on slow internet connections after a few seconds).
Please complete the anti-spam verification




user contributions licensed under cc by-sa 3.0 with attribution required

Your rights
...